Contradictory results when computing the Ideal class group of $mathbb{Q}(sqrt{-7})$












1












$begingroup$


I seem to have arrived at some contradictory results in computing this group, would you mind helping me resolve this?



By Sage + internet I find that it should be true that this class group is trivial.



We know that Minkowski's bound for this group is:



$$
M_k = frac{1}{2}frac{pi}{4}sqrt{7*4} approx 2.08.
$$



Therefore the only prime we need to check is $2 mathcal{O}_k$. Notice that $x^2 + 7 equiv_2 x^2 + 1 equiv_2 (x+1)^2$, which implies that it is totally ramified. So then we have to consider $mathfrak{p}_2 = (2, sqrt{-7}+1)$. We know that $mathfrak{p}_2^2 = (2)$, which implies that we know that the order of the ideal class group is less than or equal to 2. So then we consider $(2, sqrt{-7}+1)$ and want to show that this is principle (otherwise sage/internet is wrong).



Suppose we have some element $z$ such that $(z) = (2, 1 + sqrt{-7})$. Then it must be true that $N(z) mid N(2)$ and $N(z) mid N(1+sqrt{-7}) implies N(z) mid 4, 8 implies N(z)$ is one of $1, 2, 4$. However, since $N(a+bsqrt{-7}) = a^2 + 7b^2$, we see that it is impossible for it to take on the values $2$ and $4$, therefore if such $z$ exists it must be $1$.



However, by this answer on stack exchange, such a $z$ is not possible!



Therefore I must conclude that the class group is isomorphic to $mathbb{Z}/2mathbb{Z}$, but this is clearly not true.



Could you point out my error?










share|cite|improve this question











$endgroup$












  • $begingroup$
    Minkowski bound say we have to check ideals of norm $le 2$, that is $(1)$ and the possible prime ideals above $(2)$. Indeed $(2) =(frac{1+sqrt{-7}}{2})^2$ is a product of principal ideals so the ideal class group is trivial.
    $endgroup$
    – reuns
    Dec 13 '18 at 0:12










  • $begingroup$
    I see now, I was trying to factor in $Z[sqrt{-7}]$ but the ring of integers is larger than that by @RicardoBuring.
    $endgroup$
    – TrostAft
    Dec 13 '18 at 0:14
















1












$begingroup$


I seem to have arrived at some contradictory results in computing this group, would you mind helping me resolve this?



By Sage + internet I find that it should be true that this class group is trivial.



We know that Minkowski's bound for this group is:



$$
M_k = frac{1}{2}frac{pi}{4}sqrt{7*4} approx 2.08.
$$



Therefore the only prime we need to check is $2 mathcal{O}_k$. Notice that $x^2 + 7 equiv_2 x^2 + 1 equiv_2 (x+1)^2$, which implies that it is totally ramified. So then we have to consider $mathfrak{p}_2 = (2, sqrt{-7}+1)$. We know that $mathfrak{p}_2^2 = (2)$, which implies that we know that the order of the ideal class group is less than or equal to 2. So then we consider $(2, sqrt{-7}+1)$ and want to show that this is principle (otherwise sage/internet is wrong).



Suppose we have some element $z$ such that $(z) = (2, 1 + sqrt{-7})$. Then it must be true that $N(z) mid N(2)$ and $N(z) mid N(1+sqrt{-7}) implies N(z) mid 4, 8 implies N(z)$ is one of $1, 2, 4$. However, since $N(a+bsqrt{-7}) = a^2 + 7b^2$, we see that it is impossible for it to take on the values $2$ and $4$, therefore if such $z$ exists it must be $1$.



However, by this answer on stack exchange, such a $z$ is not possible!



Therefore I must conclude that the class group is isomorphic to $mathbb{Z}/2mathbb{Z}$, but this is clearly not true.



Could you point out my error?










share|cite|improve this question











$endgroup$












  • $begingroup$
    Minkowski bound say we have to check ideals of norm $le 2$, that is $(1)$ and the possible prime ideals above $(2)$. Indeed $(2) =(frac{1+sqrt{-7}}{2})^2$ is a product of principal ideals so the ideal class group is trivial.
    $endgroup$
    – reuns
    Dec 13 '18 at 0:12










  • $begingroup$
    I see now, I was trying to factor in $Z[sqrt{-7}]$ but the ring of integers is larger than that by @RicardoBuring.
    $endgroup$
    – TrostAft
    Dec 13 '18 at 0:14














1












1








1





$begingroup$


I seem to have arrived at some contradictory results in computing this group, would you mind helping me resolve this?



By Sage + internet I find that it should be true that this class group is trivial.



We know that Minkowski's bound for this group is:



$$
M_k = frac{1}{2}frac{pi}{4}sqrt{7*4} approx 2.08.
$$



Therefore the only prime we need to check is $2 mathcal{O}_k$. Notice that $x^2 + 7 equiv_2 x^2 + 1 equiv_2 (x+1)^2$, which implies that it is totally ramified. So then we have to consider $mathfrak{p}_2 = (2, sqrt{-7}+1)$. We know that $mathfrak{p}_2^2 = (2)$, which implies that we know that the order of the ideal class group is less than or equal to 2. So then we consider $(2, sqrt{-7}+1)$ and want to show that this is principle (otherwise sage/internet is wrong).



Suppose we have some element $z$ such that $(z) = (2, 1 + sqrt{-7})$. Then it must be true that $N(z) mid N(2)$ and $N(z) mid N(1+sqrt{-7}) implies N(z) mid 4, 8 implies N(z)$ is one of $1, 2, 4$. However, since $N(a+bsqrt{-7}) = a^2 + 7b^2$, we see that it is impossible for it to take on the values $2$ and $4$, therefore if such $z$ exists it must be $1$.



However, by this answer on stack exchange, such a $z$ is not possible!



Therefore I must conclude that the class group is isomorphic to $mathbb{Z}/2mathbb{Z}$, but this is clearly not true.



Could you point out my error?










share|cite|improve this question











$endgroup$




I seem to have arrived at some contradictory results in computing this group, would you mind helping me resolve this?



By Sage + internet I find that it should be true that this class group is trivial.



We know that Minkowski's bound for this group is:



$$
M_k = frac{1}{2}frac{pi}{4}sqrt{7*4} approx 2.08.
$$



Therefore the only prime we need to check is $2 mathcal{O}_k$. Notice that $x^2 + 7 equiv_2 x^2 + 1 equiv_2 (x+1)^2$, which implies that it is totally ramified. So then we have to consider $mathfrak{p}_2 = (2, sqrt{-7}+1)$. We know that $mathfrak{p}_2^2 = (2)$, which implies that we know that the order of the ideal class group is less than or equal to 2. So then we consider $(2, sqrt{-7}+1)$ and want to show that this is principle (otherwise sage/internet is wrong).



Suppose we have some element $z$ such that $(z) = (2, 1 + sqrt{-7})$. Then it must be true that $N(z) mid N(2)$ and $N(z) mid N(1+sqrt{-7}) implies N(z) mid 4, 8 implies N(z)$ is one of $1, 2, 4$. However, since $N(a+bsqrt{-7}) = a^2 + 7b^2$, we see that it is impossible for it to take on the values $2$ and $4$, therefore if such $z$ exists it must be $1$.



However, by this answer on stack exchange, such a $z$ is not possible!



Therefore I must conclude that the class group is isomorphic to $mathbb{Z}/2mathbb{Z}$, but this is clearly not true.



Could you point out my error?







abstract-algebra algebraic-number-theory ideal-class-group






share|cite|improve this question















share|cite|improve this question













share|cite|improve this question




share|cite|improve this question








edited Dec 13 '18 at 0:44









André 3000

12.7k22243




12.7k22243










asked Dec 12 '18 at 23:50









TrostAftTrostAft

423412




423412












  • $begingroup$
    Minkowski bound say we have to check ideals of norm $le 2$, that is $(1)$ and the possible prime ideals above $(2)$. Indeed $(2) =(frac{1+sqrt{-7}}{2})^2$ is a product of principal ideals so the ideal class group is trivial.
    $endgroup$
    – reuns
    Dec 13 '18 at 0:12










  • $begingroup$
    I see now, I was trying to factor in $Z[sqrt{-7}]$ but the ring of integers is larger than that by @RicardoBuring.
    $endgroup$
    – TrostAft
    Dec 13 '18 at 0:14


















  • $begingroup$
    Minkowski bound say we have to check ideals of norm $le 2$, that is $(1)$ and the possible prime ideals above $(2)$. Indeed $(2) =(frac{1+sqrt{-7}}{2})^2$ is a product of principal ideals so the ideal class group is trivial.
    $endgroup$
    – reuns
    Dec 13 '18 at 0:12










  • $begingroup$
    I see now, I was trying to factor in $Z[sqrt{-7}]$ but the ring of integers is larger than that by @RicardoBuring.
    $endgroup$
    – TrostAft
    Dec 13 '18 at 0:14
















$begingroup$
Minkowski bound say we have to check ideals of norm $le 2$, that is $(1)$ and the possible prime ideals above $(2)$. Indeed $(2) =(frac{1+sqrt{-7}}{2})^2$ is a product of principal ideals so the ideal class group is trivial.
$endgroup$
– reuns
Dec 13 '18 at 0:12




$begingroup$
Minkowski bound say we have to check ideals of norm $le 2$, that is $(1)$ and the possible prime ideals above $(2)$. Indeed $(2) =(frac{1+sqrt{-7}}{2})^2$ is a product of principal ideals so the ideal class group is trivial.
$endgroup$
– reuns
Dec 13 '18 at 0:12












$begingroup$
I see now, I was trying to factor in $Z[sqrt{-7}]$ but the ring of integers is larger than that by @RicardoBuring.
$endgroup$
– TrostAft
Dec 13 '18 at 0:14




$begingroup$
I see now, I was trying to factor in $Z[sqrt{-7}]$ but the ring of integers is larger than that by @RicardoBuring.
$endgroup$
– TrostAft
Dec 13 '18 at 0:14










1 Answer
1






active

oldest

votes


















3












$begingroup$

The ring of integers of $mathbb{Q}(sqrt{-7})$ is not $mathbb{Z}[sqrt{-7}]$ but bigger, because $-7 equiv 1 pmod 4$.






share|cite|improve this answer









$endgroup$









  • 2




    $begingroup$
    Oh of course, it's actually $mathbb{Z}[ (1 + sqrt{-7})/2 ]$, and then we immediately see it. That was crazy of me, thanks.
    $endgroup$
    – TrostAft
    Dec 13 '18 at 0:12













Your Answer





StackExchange.ifUsing("editor", function () {
return StackExchange.using("mathjaxEditing", function () {
StackExchange.MarkdownEditor.creationCallbacks.add(function (editor, postfix) {
StackExchange.mathjaxEditing.prepareWmdForMathJax(editor, postfix, [["$", "$"], ["\\(","\\)"]]);
});
});
}, "mathjax-editing");

StackExchange.ready(function() {
var channelOptions = {
tags: "".split(" "),
id: "69"
};
initTagRenderer("".split(" "), "".split(" "), channelOptions);

StackExchange.using("externalEditor", function() {
// Have to fire editor after snippets, if snippets enabled
if (StackExchange.settings.snippets.snippetsEnabled) {
StackExchange.using("snippets", function() {
createEditor();
});
}
else {
createEditor();
}
});

function createEditor() {
StackExchange.prepareEditor({
heartbeatType: 'answer',
autoActivateHeartbeat: false,
convertImagesToLinks: true,
noModals: true,
showLowRepImageUploadWarning: true,
reputationToPostImages: 10,
bindNavPrevention: true,
postfix: "",
imageUploader: {
brandingHtml: "Powered by u003ca class="icon-imgur-white" href="https://imgur.com/"u003eu003c/au003e",
contentPolicyHtml: "User contributions licensed under u003ca href="https://creativecommons.org/licenses/by-sa/3.0/"u003ecc by-sa 3.0 with attribution requiredu003c/au003e u003ca href="https://stackoverflow.com/legal/content-policy"u003e(content policy)u003c/au003e",
allowUrls: true
},
noCode: true, onDemand: true,
discardSelector: ".discard-answer"
,immediatelyShowMarkdownHelp:true
});


}
});














draft saved

draft discarded


















StackExchange.ready(
function () {
StackExchange.openid.initPostLogin('.new-post-login', 'https%3a%2f%2fmath.stackexchange.com%2fquestions%2f3037406%2fcontradictory-results-when-computing-the-ideal-class-group-of-mathbbq-sqrt%23new-answer', 'question_page');
}
);

Post as a guest















Required, but never shown

























1 Answer
1






active

oldest

votes








1 Answer
1






active

oldest

votes









active

oldest

votes






active

oldest

votes









3












$begingroup$

The ring of integers of $mathbb{Q}(sqrt{-7})$ is not $mathbb{Z}[sqrt{-7}]$ but bigger, because $-7 equiv 1 pmod 4$.






share|cite|improve this answer









$endgroup$









  • 2




    $begingroup$
    Oh of course, it's actually $mathbb{Z}[ (1 + sqrt{-7})/2 ]$, and then we immediately see it. That was crazy of me, thanks.
    $endgroup$
    – TrostAft
    Dec 13 '18 at 0:12


















3












$begingroup$

The ring of integers of $mathbb{Q}(sqrt{-7})$ is not $mathbb{Z}[sqrt{-7}]$ but bigger, because $-7 equiv 1 pmod 4$.






share|cite|improve this answer









$endgroup$









  • 2




    $begingroup$
    Oh of course, it's actually $mathbb{Z}[ (1 + sqrt{-7})/2 ]$, and then we immediately see it. That was crazy of me, thanks.
    $endgroup$
    – TrostAft
    Dec 13 '18 at 0:12
















3












3








3





$begingroup$

The ring of integers of $mathbb{Q}(sqrt{-7})$ is not $mathbb{Z}[sqrt{-7}]$ but bigger, because $-7 equiv 1 pmod 4$.






share|cite|improve this answer









$endgroup$



The ring of integers of $mathbb{Q}(sqrt{-7})$ is not $mathbb{Z}[sqrt{-7}]$ but bigger, because $-7 equiv 1 pmod 4$.







share|cite|improve this answer












share|cite|improve this answer



share|cite|improve this answer










answered Dec 13 '18 at 0:09









Ricardo BuringRicardo Buring

1,4711334




1,4711334








  • 2




    $begingroup$
    Oh of course, it's actually $mathbb{Z}[ (1 + sqrt{-7})/2 ]$, and then we immediately see it. That was crazy of me, thanks.
    $endgroup$
    – TrostAft
    Dec 13 '18 at 0:12
















  • 2




    $begingroup$
    Oh of course, it's actually $mathbb{Z}[ (1 + sqrt{-7})/2 ]$, and then we immediately see it. That was crazy of me, thanks.
    $endgroup$
    – TrostAft
    Dec 13 '18 at 0:12










2




2




$begingroup$
Oh of course, it's actually $mathbb{Z}[ (1 + sqrt{-7})/2 ]$, and then we immediately see it. That was crazy of me, thanks.
$endgroup$
– TrostAft
Dec 13 '18 at 0:12






$begingroup$
Oh of course, it's actually $mathbb{Z}[ (1 + sqrt{-7})/2 ]$, and then we immediately see it. That was crazy of me, thanks.
$endgroup$
– TrostAft
Dec 13 '18 at 0:12




















draft saved

draft discarded




















































Thanks for contributing an answer to Mathematics Stack Exchange!


  • Please be sure to answer the question. Provide details and share your research!

But avoid



  • Asking for help, clarification, or responding to other answers.

  • Making statements based on opinion; back them up with references or personal experience.


Use MathJax to format equations. MathJax reference.


To learn more, see our tips on writing great answers.




draft saved


draft discarded














StackExchange.ready(
function () {
StackExchange.openid.initPostLogin('.new-post-login', 'https%3a%2f%2fmath.stackexchange.com%2fquestions%2f3037406%2fcontradictory-results-when-computing-the-ideal-class-group-of-mathbbq-sqrt%23new-answer', 'question_page');
}
);

Post as a guest















Required, but never shown





















































Required, but never shown














Required, but never shown












Required, but never shown







Required, but never shown

































Required, but never shown














Required, but never shown












Required, but never shown







Required, but never shown







Popular posts from this blog

Bundesstraße 106

Le Mesnil-Réaume

Ida-Boy-Ed-Garten